LSAT and Law School Admissions Forum

Get expert LSAT preparation and law school admissions advice from PowerScore Test Preparation.

User avatar
 Dave Killoran
PowerScore Staff
  • PowerScore Staff
  • Posts: 5862
  • Joined: Mar 25, 2011
|
#55803
Complete Question Explanation
(The complete setup for this game can be found here: lsat/viewtopic.php?t=23743)

The correct answer choice is (B)

As briefly mentioned in the game setup, only two of the clients can possibly meet first, at 1:00. Let’s take a moment to review this inference further.

As established in the discussion of the third rule, the first three rules can be connected together to create the following chain:


pt45_d04_g1_q2.png


From this chain we can infer that T, W, and Y can never be first since each must come after S (although, for the purposes of this question, W is irrelevant since W is not an actual client).

From the fourth rule, we can infer that R can never be first since R must always meet with Patterson after U meets with Patterson. Consequently, we have eliminated R, T, and Y from meeting with Patterson at 1:00. This leaves only S and U as clients who could possibly meet with Patterson at 1:00, and thus “two” is the correct answer.

Answer choice (A), (C), (D), and (E): As discussed above, these answer choices must be incorrect because they do not state the maximum number of clients that could meet with Patterson at 1:00.

Answer choice (B): This is the correct answer choice.
You do not have the required permissions to view the files attached to this post.

Get the most out of your LSAT Prep Plus subscription.

Analyze and track your performance with our Testing and Analytics Package.